URGENT!! ILL GIVE
BRAINLIEST!!!! AND 100 POINTS!!!

URGENT!! ILL GIVEBRAINLIEST!!!! AND 100 POINTS!!!

Answers

Answer 1
1. Vertical
2. Vertical
3. Alternating exterior
4. Corresponding
5. Supplementary

Related Questions

(6,- 8); 2x + 3y = 6
A. Write in slope intercept form
B. Write equation in the form Ax+By=C

Answers

Let: y=mx+b (intercept form)
And you can change it into B part

Work out the circumference of this circle.
Take pie T to be 3.142 and give your answer to 1 decimal place.
4 cm

Answers

Answer:

25.1 cm

Step-by-step explanation:

4 * 2 * 3.142 = 25.136 = 25.1 cm

help by the end of the week.

Answers

Answer:

1 ) Slope of the line

[tex]m = 2[/tex]

2 ) Equation of the line

[tex]y=2x-2[/tex]

3 ) Values of a and b

a = 5

b = 6

4 ) What is the y-coordinate when?

The y-coordinate is the vertical position of a point in terms of distance and direction along the y-axis.

In this graph, when x = 2 they y-coordinate is also 2 so it is 2 units from the y-axis.

Hope this helps! :)

please help me with this im struggling :(

Answers

The equation that best represent the relationship on the scatter plot is y = 6 / 5 x + 18 / 5.

How to find the equation of a scatter plot?

The line of best fit is used to express a relationship in a scatter plot of different data points.

The line of best fit (or trendline) is an educated guess about where a linear equation might fall in a set of data plotted on a scatter plot.

Therefore, the line of best fit will pass through (2, 6)(7, 12) and (12, 18).

Therefore, the equation of the line can be represented as follows;

y = mx + b

where

m = slopeb = y-intercept

Therefore,

m = 12 - 6 / 7 - 2

m = 6 / 5

Let's find y-intercept using (2, 6)

6 = 6 / 5(2) + b

6 - 12 / 5 = b

b = 18 / 5

Therefore, the equation is y = 6 / 5 x + 18 / 5

learn more on equation here: https://brainly.com/question/12120058

#SPJ1

Find the maximum value of the function =z+8x3y subject to the following constraints.
≤+xy15≤+2xy18≥x0≥y0
Note that the ALEKS graphing calculator can be used to make computations easier.

Answers

The maximum value of the function z is 120

How to determine the maximum value of the function?

The objective function is given as

z = 8x + 3y

While the constraints are given as

x + y ≤ 15

2x + y ≤ 18

x ≥ 0 and y ≥ 0

To determine the feasible region, we simply plot the graphs of x + y ≤ 15 and 2x + y ≤ 18

And we subset the graph to where x ≥ 0 and y ≥ 0

From the graph, we have the feasible coordinates to be

(x, y) = (0, 18), (3, 12) and (15, 0)

Substitute (x, y) = (0, 18), (3, 12) and (15, 0) in z = 8x + 3y

z = 8(0) + 3(18) = 54

z = 8(3) + 3(12) = 60

z = 8(15) + 3(0) = 120

Hence, the maximum value is 120

Read more about objective functions at

https://brainly.com/question/14309521

#SPJ1

Please help me this is a chaos, im giving 20 points!!

Answers

The constant of proportionality that relates the quantities x and y is 2.25.

An equation that relates the quantities is, y = 2.25x.

What is constant of proportionality?

The ratio connecting two given numbers in what is known as a proportional relationship is the constant of proportionality. Constant ratio, constant rate, unit rate, constant of variation, and even rate of change are other names for the constant of proportionality.

1) From the given table,

   4.5 / 2 = 2.25

   11.25 / 5 = 2.25

   15.75 / 7 = 2.25

2) The constant of proportionality that relates the quantities x and y is,

y / x = 2.25

3)  An equation that relates the quantities is,

y = kx,

where, k is constant of proportionality.

Here, k = 2.25

Therefore,

y = 2.25x

To know more about the constant of proportionality, click on the link

https://brainly.com/question/28413384

#SPJ1

A sweater has an original price of $65. It is on sale for 25% off. What is the amount of the discount?

Answers

Answer:

16.25

65 x 0.25 = 16.25

If another type of question the total price would be 48.75 :)

If f is a linear function for which f(4) = −13 and f(−4) = 19, find f(5). f(5) =

Answers

The value of the function f(5) is - 17.

How to solve linear function?

A linear function is a function that represents a straight line on the coordinate plane.

A linear function in slope intercept form is as follows:

y = mx + b

where

m = slopeb = y-intercept

Therefore, for the linear function,  f(4) = −13 and  f(−4) = 19.

f(5) can be found as follows;

-13  = 4m + b

19 = -4m + b

2b = 6

b = 6  /2

b = 3

-13 - 3 = 4m

-16 = 4m

m = -4

Hence,

f(5) = -4(5) + 3

f(5) = -20 + 3

f(5) = - 17

learn more on linear function here:https://brainly.com/question/17162759

#SPJ1

Researchers who examined health records of thousands of males found that men who died of myocardial infarction​ (heart attack) tended to be shorter than men who did not.
1) Is this an​ experiment? If​ not, what kind of study is​ it?
​2) Is it correct to conclude that shorter men are at higher risk of dying from a heart​ attack? Explain.
A. Yes, since the result is considered to be statistically significant.
B. ​No, since there may be confounding variables interfering with the​ treatment, such as patient age or weight.
C. ​No, since it cannot be determined which of the subjects will die from heart attacks in the future.
D. ​No, since there may be lurking​ variables, such as patient age or weight.

Answers

1. It is an experiment and the kind of study is the case - control study.

2. We can't conclude that shorter men are at higher risk of dying from a heart attack ad D. No, since there may be lurking variables, such as patient age or weight.

What is a case - control study?

It should be noted that a case control study is an observational study where there are two groups that are compared based on certain attribute.

It should also be noted that it's an experiment as a research was made to get some needed information about heart attack.

Lastly, we can't conclude that shorter men are at higher risk of dying from a heart attack as some other variables are missing from the information provided.

Learn more about experiments on:

brainly.com/question/17274244

#SPJ1

What is 6/21 + 6/7? Please help with question

Answers

Answer: 8/7 Or In Mixed Number Form 1 1/7

Step-by-step explanation: Get the common denominator which is 21 because you get 6/7 and multiply the number and denominator by 3. You will get 18/21. Now you add. 18/21 + 6/21 = 24/21. Simplify them by the greatest common factor and so I divide by 3 and get 8/7.

Hoped This Helped!

Answer:

Step-by-step explanation:

6/21 + 6/7

6/21 + 18/21

   24/21   = 1  1 / 7

6 x 3      18

____ =  __

7 x 3      21

- You need common denominators to add fractions together!!

Find the volume of a right circular cone-shaped building with a radius of 3cm and a height of 14 cm.

Answers

The volume of the right circular cone is, 131.95 cubic centimeter.

What is cone?

A cone is a smooth-tapering three-dimensional geometric object with a flat base and an apex or vertex. The base of a cone is made up of all the points on a plane other than the apex, and the apex is connected to all the other points on the base by a series of line segments, half-lines, or lines.

Here, the radius, r = 3 cm, and height h = 14 cm.

We have to find the volume of right circular cone.

Since,

Volume = [tex]\pi r^2\frac{h}{3}[/tex]

[tex]V = \pi (3)^3(\frac{14}{3})\\ V = \pi (3)(14)\\V = 42\pi \\V = 131.95 cm^3[/tex]

Hence, the volume of the right circular cone is, 131.95 cubic centimeter.

To know more about the cone, click on the link

https://brainly.com/question/1082469

#SPJ1

For the parallelogram, if m / 2 = 4x - 20 and m /4 = 3x - 11, find m / 1. The diagram is not drawn to scale.

Answers

For a parallelogram having (∠1 and ∠3) and (∠2 and ∠4) as the two pairs of opposite angles, if [∠2 = (4x - 20)] and [∠4 = (3x - 11)], then the measure of ∠1 will be 164°.

As per the question statement, a parallelogram has (∠1 and ∠3) and         (∠2 and ∠4) as the two pairs of opposite angles, and [∠2 = (4x - 20)] and [∠4 = (3x - 11)],

Then, we are required to calculate the measure of ∠1.

To solve this question, first we will need to know about two important properties of a parallelogram,

(a) The sum of all the four interior angles of a parallelogram is equal to 360°,

(b) Opposite angles of a parallelogram are equal.

Now since our concerned parallelogram has (∠1 and ∠3) and (∠2 and ∠4) as the two pairs of opposite angles,

Therefore, (∠1 = ∠3) and (∠2 = ∠4),

And [(4x - 20) = (3x - 11)]...[∵(∠2 = ∠4)]

Or, [(4x - 3x) = (20 - 11)]

Or, [x = 9]

Therefore, [∠2 = ∠4 = (4x - 20) = 16°].

Now, [(∠1 + ∠2 + ∠3 + ∠4) = 360°]

Or, [(∠1 + ∠2 + ∠1 + ∠2) = 360°]...[∵(∠1 = ∠3) and (∠2 = ∠4)]

Or, [{(∠1 + ∠1 ) + (∠2 + ∠2)} = 360°]

Or, [(2∠1 + 2∠2) = 360°]

Or, [2∠1 = {360 - (2 * 16)}°]

Or, [2∠1 = (360 - 32)°]

Or, [2∠1 = 328°]

Or, [∠1 = (328/2)°]

Or, [∠1 = 164°]

That is, for a parallelogram having (∠1 and ∠3) and (∠2 and ∠4) as the two pairs of opposite angles, if [∠2 = (4x - 20)] and [∠4 = (3x - 11)], then the measure of ∠1 will be 164°

To learn more about Parallelograms and angles, click on the link below.

https://brainly.com/question/19310760

#SPJ1

6. ΔABC ≅ ΔYWZ
x=___ AC= ____ T=____ WZ= ___ WY=____
AB=____ m 7. ΔABC ≅ Δ ZYW
X=___ YZ=____ T=____ WZ=___ WY=___

Answers

6) The missing angles and lengths are; x = 14, T = 14, AB = WY = 60

7) The missing angles and lengths are; X = 7, ∠A = ∠Z =31°, T = 9, YZ = AB = 28

How to interpret congruent triangles?

6) We are told that;

ΔABC ≅ ΔYWZ

That means triangle ABC is congruent to triangle YWZ

Thus;

∠C ≅ ∠Z

5x - 4 = 66

5x = 70

x = 14

Similarly, AB = WY

5T - 10 = 4T + 4

5T - 4T = 10 + 4

T = 14

AB = 5(14) - 10

AB = 60 = WY

7) We are told that;

ΔABC ≅ ΔZYW

That means triangle ABC is congruent to triangle ZYW

Thus;

∠A ≅ ∠Z

5X - 4 = 3X + 10

2X = 14

X = 7

∠A = 5(7) - 4 = 31° = ∠Z

Similarly;

2T + 10 = 4T - 8

T = 18/2

T = 9

YZ = 4(9) - 8

YZ = 28 = AB

Read more about congruent triangles at; https://brainly.com/question/7727792

#SPJ1

Determine which set of side measurements could be used to form a right triangle.

6, 12, 60
4, 5, 7
square root of 27 comma square root of 35 comma 62
square root of 12 comma 3 comma square root of 21

Answers

Answer:

square root of 12, 3,  square root of 21

Step-by-step explanation:

a

3.5 in

b

3 in

c (hypotenuse)

4.60977 in

Area

5.25

in²

The set of side measurements  √12, 3, and √21 could be used to form a right triangle. which is the correct answer would be option (D).

What is Pythagoras's theorem?

Pythagoras's theorem states that a right-angled triangle, the square of one side is equal to the sum of the squares of the other two sides.

We have to apply Pythagoras's theorem in all the given options to find the right triangle.

As per option (A),

6, 12, 60

6² + 12² = 60²

36 + 144 = 3600

180 = 3600

This equation is not true.

As per option (B),

4, 5, 7

4² + 5² = 7²

16 + 25 = 49

41 = 49

This equation is not true.

As per option (C),

√27, √35, 62

(√27)² + (√35)² = 62²

27 + 35 = 3844

This equation is not true.

As per option (D),

√12, 3, √21

(√12)² + (3)² = (√21)²

12 + 9 = 21

21 = 21

The above equation is true and satisfies Pythagoras's theorem which means these measurements could be used to form a right triangle.

Hence, the correct answer would be an option (D).

Learn more about Pythagoras's theorem here:

brainly.com/question/343682

#SPJ1

If you took out a loan for $10,000 with a 2% interest rate & it was for 18 months. How much interest would pay?

Answers

The amount of interest he would pay for a  loan of $10,000 with a 2% interest rate and for 18 months is $300

Amount of money taken as a loan = $10000

The rate of interest = 2%

The time period of interest is 18 months = 1.5 year

Interst = prt/100

= 10000x 2 x 1.5/100

= 300

Therefore, the amount of interest he would pay for a  loan of $10,000 with a 2% interest rate and for 18 months is $300

To learn more about interest refer here

https://brainly.com/question/26011426

#SPJ9

The longest runway at an airport has the shape of a rectangle and an area of 2,304,700 square feet. This runway is 190 feet wide. How long is the​ runway?

Answers

The runway is 12,130 feet long.

To find are you have to times the two sides, in this case it's 190 X 12,130. To find the length of one of these sides you must divide the area by one known side, in this case 2,304,700/190, which means the answer is 12,130 feet

use synthetic division to find the quotient and remainder of the following. Write your answer on the form of ​

Answers

Answer:

Step-by-step explanation:

1. x^2 + 4x + 5/x-1

2. x^3-3x^2 + 9x-2 + 3/x+3

3. 2x^2 - 2x+5- 14/3x+1

4.3x2^2- 4x+3 - 13/ 2x-1

5. 3x^2 + 6x + 7 + 12/2x-3

For some particular website, the password must start with one of the letters W, X, Y, or Z. Then there must be 5 more charaters, and each of those characters may be a numeric digit or a letter of the alphabet. How many different passwords are possible?

Answers

241,864,704  many different passwords are possible.

The total characters of the password is six in which it is given that first character should start with W, X, Y or Z. So the first character can be filled in 4 ways.

For next five characters it can be any letter of the alphabet or any numeric digit so it will be 26 + 10 = 36.

So second, third, fourth, fifth and sixth space can be taken by 36 characters.

Therefore the total no. of passwords will be 4 * 36 * 36 * 36 * 36 * 36 that is equal to 241,864,704.

To solve more questions like this

https://brainly.com/question/14767366

#SPJ1

A car rental company charges an initial fee plus an additional fee for each mile driven. The charge depends on the type of car: economy or luxury.
The charge E (in dollars) to rent an economy car is given by the function E=0.70M+11.95, where M is the number of miles driven. The charge L (in dollars)
to rent a luxury car is given by the function L-1.25M+17.30.
Let C be how much more it costs to rent a luxury car than an economy car (in dollars). Write an equation relating C to M. Simplify your answer as much as
possible.

Answers

The equation to find rent difference C from given values of E and L is 1.75M + 3.25. After adding the values of E and L.

How to calculate cost difference ?

A mathematical equation is a formula that uses the equals sign to represent the equality of two expressions. The meanings of the word equation and its translations into various languages can vary slightly.

Finding the values of the variables that result in the equality is the first step in solving an equation with variables.

Let E represent Economy

Let L represent Luxury

Let M be the number of miles driven

Let C be how much it cost to rent a luxury car than economy

E = -0.70M + 11.95

L = 1.25M + 17.30

C = L - E

C =(1.25M + 17.30) - (-0.70M + 11.95)

C = 1.25M + 17.30 + 0.70M - 11.95

collect like terms

C = 1.25M + 0.70M + 18.20 - 14.95

C = 1.95M + 3.25

The rent difference between luxury car and economy car is 1.95M +3.25 .

To learn more about calculating rent refer :

https://brainly.com/question/28583996

#SPJ13

Solve the formula for t.

V=6πrt+7πr*2

t=

Answers

The unknown variable t has a value of t = (V - 7πr²)/6πr

How to determine the unknown variable?

From the question, we have the following parameters that can be used in our computation:

Formula; V = 6πrt+7πr*2

Rewrite the formula properly

So, we have the following representation

V = 6πrt + 7πr²

Subtract the expression 7πr² from both sides of the equation

So, we have

V - 7πr² = 6πrt + 7πr² - 7πr²

Evaluate the difference

V - 7πr² = 6πrt

So, we have

6πrt/6πr = (V - 7πr²)/6πr

Evaluate

t = (V - 7πr²)/6πr

Hence, the value is t = (V - 7πr²)/6πr


Read more about subject of formula at

https://brainly.com/question/657646

#SPJ1

Answer:

[tex]t = \dfrac{V-7\pi r^2}{6\pi r}[/tex]

Step-by-step explanation:

V = 6πrt + 7πr²

Subtract the term without t from both sides.

V - 7πr² = 6πrt

Now you have a single term with t on the right side and two terms without t on the left side.

Switch sides.

6πrt = V - 7πr²

Divide both sides by 6πr.

[tex] t = \dfrac{V - 7\pi r^2}{6\pi r} [/tex]

Find the slope of the line that passes through the following points: (−4, 3) and (5, 4).

Answers

Answer:

The slope would be 1/9.

Step-by-step explanation:

From 3 to 4 it would be 1. From -4 to 5 it would be 9, the equation for slope is y/x so it would be 1/9.

solution

slope M =(Y' - Y)/(x' - x)

Indicated by (-4, 3) and (5 , 4)

x = -4 , x' = 5 , y = 3 , y' = 4

M = (4 - 3)/( 5 + 4)

M = 1/9 Ans

Which number is an integer?
2.3
0
-3/4
pi

Answers

0 is an integer number.

A number without a decimal or fractional element is known as an integer, which encompasses both positive and negative numbers, including zero.

i.e. the set of integers is

Z = {... -7, -6, -5, -4, -3, -2, -1, 0, 1, 2, 3, ...}

The integers do not contain any fraction or rational number.

Here,

2.3 is a fraction number so it is not an integer.

And -3/4 is a rational number and rational numbers are not integers.

And pi is irrational number

So,

This three numbers are not an integer

Now,

0 is an integer as per the definition of integers.

Hence,

0 is the integer number

To know more about integers refer to :

https://brainly.com/question/929808

#SPJ1

5. Rewrite the expression as a single logarithm. (1 point)

Answers

We can rewrite the given logarithmic expression as [tex]ln(\frac{\sqrt{x}(x-1)^3 }{\sqrt[3]({x+1})^2 })[/tex].

So option (D) is the correct answer.

What is a logarithmic expression?

A logarithmic equation is one that involves the logarithm of a variable-containing expression. To solve exponential equations, first, determine whether both sides of the equation can be written as powers of the same number.

The given logarithmic expression can be expressed as

[tex]=\frac{1}{2}\ln}\left(x\right)+3\left(\ln\left(x-1\right)-\frac{2}{9}\ln\left(x+1\right)\right)[/tex]

[tex]=\frac{\ln\left(x\right)}{2}+3\left(\ln\left(x-1\right)-\frac{2}{9}\ln\left(x+1\right)\right)[/tex]

[tex]=\frac{\ln\left(x\right)}{2}+3\ln\left(x-1\right)-\frac{2\ln\left(x+1\right)}{3}[/tex]

By using the logarithmic properties, we get

[tex]=ln(\frac{\sqrt{x}(x-1)^3 }{\sqrt[3]({x+1})^2 })[/tex]

We can rewrite the given logarithmic expression as [tex]ln(\frac{\sqrt{x}(x-1)^3 }{\sqrt[3]({x+1})^2 })[/tex].

So option (D) is the correct answer.

To learn more about the logarithmic expression visit:

https://brainly.com/question/20473168

#SPJ9

1. Corey buys a Toyota Yaris for $15,000 at 4.1% interest rate for 6 years.

How much interest will Corey pay on his Yaris?

What is the total amount that will paid?

What is his monthly payment?

Answers

Answer:

a) $4089.55

b) $19089.55

c) $56.80

Step-by-step explanation:

a) (100 + 4.1)/100 = 1.041. So, 15000 × 1.041⁶ = 19089.55. Interest that will be paid: 19089.55 - 15000 = $4089.55 (2 dp).

b) $19089.55 (with interest and principal)

c) 4089.55/72 = $56.80

What would the domain and range be for a problem like this. I can’t seem to recall how to determine them

Answers

The domain and range are [−4, 1] & [−4, 5].

What are the domain and range of a graph?

Graphs can also be used to determine the domain and range of functions. Because the domain refers to the set of possible input values, the domain of a graph includes all of the input values shown on the x-axis. The range is the set of possible output values shown on the y-axis.

We have given the graph,

The domain refers to the set of possible input values, the domain of a graph consists of all the input values shown on the x-axis.

The range is the set of possible output values, which are shown on the y-axis.

We can observe that the graph extends horizontally from −4 to the right 1.  so the domain of function h is [−4, 1].

The vertical extent of the graph is 0 to –4, so the range of function h is [−4, 5].

Therefore, the domain and range are [−4, 1] & [−4, 5].

To learn more about the domain and range visit,

https://brainly.com/question/2264373

#SPJ9

A toy rocket is shot vertically into the air from a launching pad 7 feet above the ground with an initial velocity of 72 feet per second. The height h, in feet, of the rocket above the ground at t seconds after launch is given by the function h(t)=-16 t²+72 t+7. How long will it take the rocket to reach its maximum height? What is the maximum height?
The rocket reaches its maximum height at ______ second(s) after launch.
(Simplify your answer.)
The maximum height reached by the object is ______feet.
(Simplify your answer.)

Answers

The time taken for the rocket to reach its maximum height as determined from the given information is; 2.25 seconds.

The maximum height reached by the toy rocket is; 88 feet.

What is the maximum height reached by the rocket?

It follows from the task content that the maximum height reached by the rocket is to be determined.

The given function is; h (t) = -16 t² + 72t + 7.

Since it follows from evaluation that; at maximum height, the rate of change of height to time is equal to 0.

Therefore; at maximum height;

h'(t) = 0

h'(t) = -32t + 72 = 0

-32t = -72

t = -72/-32

t = 2.25 seconds.

Therefore, the time taken for the rocket to reach its maximum height is; 2.25 seconds.

Also, the maximum height reached by the rocket is at; h (2.25);

h(2.25) = -16(2.25)² + 72 (2.25) + 7

= -81 + 162 + 7

= 88.

The maximum height reached is; 88 feet.

Read more on maximum height;

https://brainly.com/question/82347

#SPJ1

Which triangle congruence postulate or theorem proves that these triangles
are congruent?

Answers

The Angle-Side-Angle Postulate (ASA) asserts that two triangles are congruent if their two angles and included sides are congruent with each other.

What is  Angle-Side-Angle  Postulate ?According to the definition of an angle side angle, two triangles are congruent to one another if two angles from one triangle and the side between them are, respectively, equal to two angles from another triangle and the side between them. If two triangles are congruent, thenOne triangle's three sides will be equivalent to its three sides, respectively.The three angles of one triangle will be equivalent to the three angles of the other, respectively.But we don't always need to know information about all the sides and angles of a triangle to be sure that two triangles are congruent. Let's use an example to illustrate ASA.Let's use an example to illustrate ASA. Take into account the triangles ABC and DEF:

We are given that,

BC = EF

∠B = ∠E

∠C = ∠F

We say that by ASA criterion: Δ ABC ≅ ΔDEF.

To Learn more About Angle-Side-Angle Postulate refer to :

https://brainly.com/question/3168048

#SPJ1

Select all the solutions to the inequality 2(7x + 1) < -26
_
A. -2
B. 9
C. -7
D. 4

Answers

Answer:X<-2

Step-by-step explanation:

7x+1<-13

7x<-13-1

7x<-14

Answer:

Step-by-step explanation:

the answer is A: x<-2

Consider the following scenario: P(E) = 0.4, P(F) = 0.5

a. Find P(E or F) if E and F are disjoint

b. Find P(E or F) if E and F are independent

Answers

Part (a)

The term "disjoint" is the same as "mutually exclusive".

If events E and F are disjoint, then we know that P(E and F) = 0. In other words, the two events have nothing in common. Therefore, it's impossible for them both to occur simultaneously.

P(E or F) = P(E) + P(F) - P(E and F)

P(E or F) = 0.4 + 0.5 - 0

P(E or F) = 0.9

Answer:    0.9

====================================================

Part (b)

If events E and F are independent, then P(E and F) = P(E)*P(F).

P(E or F) = P(E) + P(F) - P(E and F)

P(E or F) = P(E) + P(F) - P(E)*P(F)

P(E or F) = 0.4 + 0.5 - 0.4*0.5

P(E or F) = 0.4 + 0.5 - 0.2

P(E or F) = 0.9 - 0.2

P(E or F) = 0.7

Answer:   0.7

Josie has $47 left on her checking account. If she writes a check for $55, what will Josie's balance be?

Answers

If Josie has $47 left on her checking account, and she writes a check for $55 , then she takes out money from her account and ends up losing money. Thus, subtract $55 from $47 . ∴ , Josie's balance will be $−8 or $8 in debt.
Other Questions
African folklore/ Quilombo project In the end, why must everyone in Palma res be exterminated? Which descriptions from the list below accurately describe the relationshipbetween AQRS and ATUV? Check all that apply.A. CongruentB. Similarc. Same sizeD. Same sides Element X, the oxide of which has the formulo XO7 form with hydrogen a Find the relative atomic that contains 0.78% hydrogen. mass of element x In gas compound in the nucleas of the atom of element x there are 21 more neutrons than photons Find the atomic number Z of element X......Please helppppppp Which of the following refers to the nvironment acting on an individual, and the individual reacting to the environment? A. disinhibition B. reciprocal determinism C. operant conditioning D. social learning For a two-tailed test, the p-value is the probability of obtaining a value for the test statistic as. what kind of cases do not involve criminal penalties and deal with disputes among individuals or between the government and individuals? sybil has delusions and frequently hears voices that are not there. additionally, she is unable to manage her emotions and think clearly. as a result, she has no friends and cannot remain employed. given this information, she is probably suffering from what disease? When two forces act on an object in opposite directions, how should you calculate the net force? Cuando patty era pequea, siempre iba al parque con su mam. a ella le gustaba tocar y oler las flores all. cuando corra, siempre se caa y se lastimaba sus rodillas. eso dola mucho y siempre terminaba llorando. patty era una nia muy hermosa con largas pestaas y ojos de color caf. qu pasaba cuando patty corra? ella estaba muy contenta ella llevaba a su perro ella siempre se caa. ella llevaba una pelota. (50 POINTS) Why did Congress want one free state and one slave state?A) to keep the residents of both states happyB) to avoid taking a stand on slaveryC) to upset the northern statesD) to keep the Senate balanced 2. Find the slope of linear demand curve for pen, when consumer purchases 100 at Rs. 10 per pen and 50 at Rs. 15 per pen. What is the significance of the appearance of the Ghost?. 7. you want to establish today a 4-year college annuity for your young niece with the first payment starting 18 years from year from now. the initial annuity payment will be $50,000 but needs to grow by 10% per year. assume prevailing interest rates of 5%. how much must you invest today to achieve this result? which of the following represent non-coding dna important in the process of dna replication? a. fork traps b. tus c. hp loops d. 1 e. dnag If you mix 30 mL of cold water with 70 mL of hot water in a calorimeter, and the temperature of the cold water increases by 1.9C, then how much heat (in joules) did the cold water absorb (only enter the number, not units, and assume the density of water to be 1.00g/mL, and the specific heat capacity of water to be 4.184 J/g-C)? y-4x=5 1/2y=-1/6x-4systems of equations (substitution your patient is in a seated position with his feet flat on the floor. he is having difficulty lifting his toes off the floor. what muscle(s) is impaired from contracting? Your client mr. Smith has requested a reservation at eleven madison park this evening for a party of 2 at 6pm. Unfortunately, the restaurant is fully committed. As an alternative, which restaurant would you suggest? you can use the internet to research the right answer. With your group, reread the lines of "Sonnet, With Bird" numbered 11 and 12. What point does the author make by defining England in two different ways? Explain On page 77, Edwin Black asserts, "The problem with Indigenous art in general is that it's stuck in the past." How does the tension between modernity and tradition emerge throughout the narrative? Which characters seek to find a balance between honoring the past and looking toward the future? When is the attempt to do so successful?